12
$\begingroup$

I am unable to visualize any case where angular momentum and angular velocity of an object are not parallel.

$\endgroup$
4
  • 1
    $\begingroup$ Optical vortex? $\endgroup$
    – user28737
    Oct 2, 2013 at 6:44
  • 1
    $\begingroup$ Can you explain what is optical vortex and how it can be an example where angular momentum and angular velocity are not paralle $\endgroup$ Oct 2, 2013 at 9:58
  • $\begingroup$ Related $\endgroup$
    – user28737
    Oct 2, 2013 at 13:44
  • 1
    $\begingroup$ @WaqarAhmad that's actually the opposite question $\endgroup$
    – user6972
    Oct 3, 2013 at 1:23

3 Answers 3

15
$\begingroup$

In the basic discussion of angular momentum where something is rotating around a fixed symmetrical axis

$\vec{L}=\vec{r}\times\vec{p}$

reduces to

$\vec{L}=I*\vec{\omega}$

Like in this animation where each vector is colored appropriately:

animation

However angular velocity and angular momentum can have different directions in two cases: If the axis of rotation is not symmetric or the axis of rotation is moving.

Here's an example:

enter image description here

You can see that $\vec{L}=\vec{r}\times\vec{p}$ is not the same direction as $\vec{\omega}$ nor would the simplification $\vec{L}=I*\vec{\omega}$ be correct.

The position vector $\vec{r}$ is the vector between the reference point and the mass (note these problems are ignoring the mass of the rod), only in simple rotational cases like the first case is it perpendicular to $\vec{\omega}$. In a system of masses for example these vectors to the masses about a reference point can be complex. It is much easier to take the reference point as the center of mass. In each case $\vec{r}$ is the positional vector between your reference point and the mass and their composite angular momentums will superimpose (add) together.

$\endgroup$
8
  • $\begingroup$ In the second illustration direction of angular velocity and linear momentum are misleading. In calculating angular momentum, L ,don't we take r as a vector perpendicular to our reference axis? $\endgroup$ Oct 2, 2013 at 9:55
  • $\begingroup$ Corect me if I am mistaken. $\endgroup$ Oct 2, 2013 at 10:22
  • $\begingroup$ Imagine this, the dumb bell is rotating about a stick. The angular momentum is parallel to the stick. But the stick itself is inclined w.r.t another, stationary, reference axis and is rotating about that axis. The reason for using r parallel to the arms and not the stationary axis is is because r is perpendicular to the axis about which the masses are rotating. But that axis itself is rotating w.r.t another.The angular momentum vector is rotating with respect to the stationary axis. $\endgroup$
    – MarcelineH
    Oct 2, 2013 at 10:57
  • 1
    $\begingroup$ @WaqarAhmad It is a misconception that angular momentum is always about the same axis as angular velocity. Sometime this may not be possible, in these cases the angular momentum component along the axis of rotation is the product of angular velocity and moment of inertia about the given axis of rotation. $\endgroup$
    – user6972
    Oct 2, 2013 at 21:53
  • 1
    $\begingroup$ I found an answer in this paper $\endgroup$ Oct 3, 2013 at 10:18
4
$\begingroup$

Let's limit the discussion to rigid objects rotating with a fixed $\vec{\omega}$, i.e. fixed axis and constant angular velocity. Here are a couple of observations:

  1. $\vec{L}$ rotates with the rigid body. We could glue a little arrow representing $\vec{L}$ to the body, so that the arrow rotates around $\vec{w}$ at the same rate as the body.

  2. If $\vec{L}$ is parallel to $\vec{w}$, then it is unaffected by the rotational motion. Otherwise, $\vec{L}$ is changing throughout the rotational motion.

  3. If $\vec{L}$ is changing, it means an external torque is acting through the axis.

Given these observations, we can state the following:

$\vec{L}$ is not parallel to $\vec{\omega}$ if and only if a torque is needed to keep an object rotating around a fixed axis.

To get a feel for this, try holding a spinning bicycle wheel in your hands. If the wheel is slightly imbalanced, you will feel an oscillating torque from the axis - this is coming from the changes in $\vec{L}$ as it spins around $\vec{\omega}$. If the wheel is perfectly balanced, then you will feel nothing - $\vec{L}$ is parallel to $\vec{\omega}$ and is fixed throughout the rotation.

enter image description here

Here is a diagram contributed by @user6972 showing exactly how $\vec{L} = \vec{r} \times \vec{p}$ can have a different direction to $\vec{\omega}$:

enter image description here

$\endgroup$
2
$\begingroup$

The answer above gives examples but if you want to know 'HOW' it happens, consider the following equation

$$ L_i = \sum_j I_{ij} \omega_j $$

You can diagonalize the symmetric moment of inertia tensor (by rotating your basis. In that basis you get

$$ L_{i'} = \sum_{j'} \lambda_{j'} \delta_{i'j'} \omega_{j'} = \lambda_{i'} \omega_{i'} $$

and thus you see that in this basis whenever you have two or more components of $\omega$ non-zero along directions with unequal $\lambda$ you cannot have the angular momentum parallel to the angular velocity.

$\endgroup$